8.02x - Module 10.05 - Parallel RLC Circuit - Phase Angles - Impedance - Resonance

Поделиться
HTML-код
  • Опубликовано: 13 сен 2024

Комментарии • 96

  • @engestruturas
    @engestruturas 5 лет назад +33

    You made me love Parallel RLC circuit. Thank you Dr. Walter Lewin!
    Cheers from Brazil :)

  • @rishabharya3329
    @rishabharya3329 6 месяцев назад +2

    these lectures are gold!!!
    i always look upto him or HC verma ....
    if you guys know a similar teacher please let me know!

  • @maheshtripathi2799
    @maheshtripathi2799 4 года назад +24

    Studying for my JEE ADVANCED 2020 from you sir
    Thanks for helping
    Love from INDIA

    • @atharva302
      @atharva302 3 года назад +1

      Please tell jee adv aspirant here from 2021 🙏

    • @communist_amoeba3923
      @communist_amoeba3923 Год назад +1

      KVLonlyvalidforconservativefield

  • @JDBolt1958
    @JDBolt1958 4 года назад +1

    This is a band stop filter. The "anti-resonance point " makes the current low at a particular frequency and fixed LC properties.... pretty handy for a multiband ham radio antenna.However this // circuit can also function as a radio tuner. Now use the inductor from the series LRC circuit, and place it next to the parallel circuit inductor and create a transformer. substitute a variable capacitor in place of the fixed capacitor, the parallel circuit becomes a tuner, and the series circuit a band pass filter which tunes an antenna!,has taken me 43 years since college physics to master this understanding..thank you Dr. Lewin. I did get an A back in the day.....but now my eyes are fully open. I simply cannot watch this lecture enough times...awesome....simply understanding the circuit by understanding the components limiting values , Faraday's law applies loop 2 not a poor application of KIrchhoff's voltage rule, absolutely brilliant. Phasor diagram insight, leaving out Mr. Euler's equations for now.......to focus the ideas...stunning

  • @vijayr1485
    @vijayr1485 7 лет назад +32

    At 5:28, you get done writing the equation for Current through the solenoid as a function of time: (emf/w)*sin(wt) -- the integral of the equation 2. But based on the equation, shouldn't it be divided by an L term to get rid of it on the right side.

  • @nybble
    @nybble 3 года назад +3

    I was sitting here watching, at about 27 minutes about to go... No wait, you need to take the 2nd derivative to know if that's a maxima or minima! You only found a turning point!

  • @slendrmusic
    @slendrmusic 3 года назад +1

    I had trouble with this at school, but this made it look so obvious wtf. Hope you get 1M subscribers soon! If all schoolchildren watched these videos, we would have a whole new generation of enthusiastic physicists.

  • @muneer332
    @muneer332 8 лет назад +3

    amazing explanation Sir....I love ur lectures...currently I am watching ur lecture 8.02. sir I had this doubt from my school days and from the books I got some answer but confused me a lot. inductor always confused me a lot. now I got the clear answer from u and I am sure that this is the correct explanation. once again thank u sir...

  • @umasrivastava4142
    @umasrivastava4142 4 года назад +2

    Monster 😂😂 11:14
    Thank you sir for bringing this 💪

  • @GHOSTrex1324
    @GHOSTrex1324 4 года назад +2

    🙏🙏 really helped me solving parallel ac circuits.

  • @franciscoaguero9028
    @franciscoaguero9028 4 года назад +1

    What an amazing professor you are sir, i think it’s the derivative of I max with respect to W that max out the function. Great video!!

  • @BuK-HumMraanG
    @BuK-HumMraanG 4 года назад

    You open my eye. Thank you Prof. (From Thailand)

  • @muditgaur2779
    @muditgaur2779 3 года назад +1

    Dhanyawad = thank you in Hindi ❤️❤️

  • @mstew7055
    @mstew7055 3 года назад +3

    Sir at 5:36 you write I_LT=epsilon*sin(omega*t)/omega... But shouldn't it be I_LT=epsilon*sin(omega*t)/(omega*L)

  • @rekhagupta8354
    @rekhagupta8354 3 года назад +2

    Sir please take the case in which L and R are in series and C is in parallel with the LR series combination. .

  • @yuchenxu1559
    @yuchenxu1559 5 лет назад +7

    10:58 I assume tan(phi) = R(wc-1/wL).

    • @jodpar2002
      @jodpar2002 4 года назад +2

      right

    • @robertnagy3942
      @robertnagy3942 2 года назад

      that's the same, he just didn't use the reactances Xl and Xc notation, just used wL and 1/wc

  • @kanadkrishanusengupta2578
    @kanadkrishanusengupta2578 6 лет назад +2

    Sir you are the best teacher in the world...

  • @RaviPrakash-oq2zz
    @RaviPrakash-oq2zz 4 года назад +4

    Awesome lecture sir, But at 5:26 shouldn't it devided by L, when you wrote integral of equation 2 🙋

  • @ahmadabohagar610
    @ahmadabohagar610 5 лет назад +2

    Lovely ♥️

  • @Gamma_Digamma
    @Gamma_Digamma 5 лет назад +3

    Mathematical demonstration that physics works....

  • @tensorbundle
    @tensorbundle 4 года назад

    best professor

  • @gagandeep-uh4pr
    @gagandeep-uh4pr Год назад

    monster was amazing

  • @HTRSdhiman
    @HTRSdhiman 21 день назад

    Aur lakshiyan lindanes , aagye?

  • @parasharisir
    @parasharisir 2 года назад

    .... there is Electric field inside the solenoid/ inductor, but in the radial direction. Yes, no electric field along the axis. ....

  • @BentHestad
    @BentHestad 5 лет назад +1

    Excellent! Thanks, Sir!

  • @ahmadsalameh7959
    @ahmadsalameh7959 4 года назад +2

    That was great, thank you very much. But I think you meant to say that you took the derivative as a function of omega and not time at 15:08 and 16:25.

    • @lecturesbywalterlewin.they9259
      @lecturesbywalterlewin.they9259  4 года назад +13

      y=f(ω) yupe should have been dy/dω. Amazing that noone pointed that out in 1996 when this was on the air at MIT 24/7 for a whole week.

    • @cricobug9778
      @cricobug9778 4 года назад

      @@lecturesbywalterlewin.they9259 professor but how will i know this yields maxima or minima because we use same method

    • @atharva1210
      @atharva1210 3 года назад

      @@cricobug9778 in this case you know that maximum current is infinity and the curve is vertical line at infinity, whereas maxima/minima is a point on curve where derivative is zero which implies a horizontal line. So the extremum we got here will definitely be the minimum and not maximum.

  • @ugursoydan8187
    @ugursoydan8187 4 года назад +1

    how we can reach the formula?(I max) which differential equation should we solve?

  • @markcavendish7148
    @markcavendish7148 7 лет назад +1

    At 4:33 you said that total instantaneous current [ through battery ] is sum of instantaneous currents through L, C, R. But in my book it is written that " The total current drawn from the supply is equal to the vector sum of the resistive, inductive and capacitive current, not the mathematical sum of the three individual branch currents, as the current flowing in resistor, inductor and capacitor are not in same phase with each other; so they cannot be added arithmetically.
    Could you please clear this ? Is the sum of rms currents through L , C , R = rms current through source or is this true for instantaneous current ? Does phasor method work for rms currents or instantaneous currents ?
    Is Kirchhoff's 2nd law not valid for instantaneous currents ?

    • @lecturesbywalterlewin.they9259
      @lecturesbywalterlewin.they9259  7 лет назад +2

      >>>>At 4:33 you said that total instantaneous current [ through battery ] is sum of instantaneous currents through L, C, R.>>>>
      That is correct. The solution to the diff eq I(t) says it all. There is ONLY one I at a given time t and that takes the phase angles into account.

  • @eee_27_harbakshsingh38
    @eee_27_harbakshsingh38 4 года назад +2

    For me Walter lewin is enisten, nicola , bohr , schrodinger etc . Respect from india

  • @varshaaruchamy9004
    @varshaaruchamy9004 Год назад

    When I did experiment- parallel resonance,the current keep decreased,but never increase.what is the reason behind it

  • @judao1383
    @judao1383 4 года назад

    Thank you sir.

  • @life_well11
    @life_well11 3 года назад

    The way he says.. But we have time to do that

  • @mukulgidney
    @mukulgidney 5 лет назад +1

    Isn't the current in the inductor supposed to lead the one in the resistor .
    We've seemed to have done the opposite here
    I'm sorry there's no mistake
    Just a bit of a misunderstanding.
    Really appreciate this video

  • @sachinrath219
    @sachinrath219 3 года назад

    when current lags the voltage,does it mean it flows in the reverse direction to the one taken in the circuit.

  • @PhysicsFunwithDharmeshKareliya
    @PhysicsFunwithDharmeshKareliya 3 года назад

    Really trivial. Enjoyable though.

  • @MuhammadRiaz-sm7yj
    @MuhammadRiaz-sm7yj 4 года назад

    Great 😎

  • @markcavendish7148
    @markcavendish7148 6 лет назад

    In resonance :
    (1) is the instantaneous current through source = instantaneous current through resistor ; ie the entire current supplied by source goes through resistor ?
    (2) instantaneous current through inductor = instantaneous current through capacitor ( but in opposite direction )? ie ,capacitor and inductor feed each other.

  • @rajarshimukherjee4844
    @rajarshimukherjee4844 4 года назад

    Why do we ignore the constant of integration while evaluating current through the inductor? Is it justified to remove the constant just because it comes to be a DC component?

  • @nomannosher8928
    @nomannosher8928 3 года назад

    I can't understand the Infinitely high C concept.

  • @ahmadeldesokey9844
    @ahmadeldesokey9844 5 лет назад

    Sir , if we have a battery with no resistance , is it possible to have an induced E-field in the battery due to time changing B-fields in the circuit ?

    • @lecturesbywalterlewin.they9259
      @lecturesbywalterlewin.they9259  5 лет назад

      make your own calculations and estimate realistic values of the self inductance of all your components. My guess is that the sum of all will be less than 10 micro H. Take it from there

  • @ahmadeldesokey9844
    @ahmadeldesokey9844 5 лет назад

    Sir , at 3:53 you wrote faraday's law for loop number 2 , is the induced E-field lies inside the driving source for this loop ?

    • @lecturesbywalterlewin.they9259
      @lecturesbywalterlewin.they9259  5 лет назад +1

      I cannot add to the clarity of this video. I suggest you watch it again and also my 8.02 lectures which cover all this.

    • @ahmadeldesokey9844
      @ahmadeldesokey9844 5 лет назад +1

      @@lecturesbywalterlewin.they9259 it may be a trivial question for you but it will be a great help if you answered such a question .

  • @janmeshsamvlogs7917
    @janmeshsamvlogs7917 3 года назад

  • @karamkashkoush9129
    @karamkashkoush9129 3 года назад

    What will happen if the voltage source frequency was set up to zero ?

    • @atharva1210
      @atharva1210 3 года назад +1

      zero frequency means a dc source aka battery. and when battery will be connected, initially capacitor will begin to charge, current will start growing in inductor and also flow throught resistor. And at steady state no current through capacitor, inductor behaves like a normal wire, and there will be a short circuit

  • @amforever8697
    @amforever8697 2 года назад

    Sir What Is Vertasium 79 Said In Electrical energy Is its right

  • @muneer332
    @muneer332 8 лет назад

    sir..what will be the voltage across the inductor if the electric field is zero..whether it is zero or.. it is the source voltage ...pls provide me an answer..thank you

    • @lecturesbywalterlewin.they9259
      @lecturesbywalterlewin.they9259  8 лет назад +2

      In case of non-conservative E fields, it's better not to talk about "voltage"or "potential difference" over a self-inductor as it depends on how you measure it. If you define potential difference as the integral of E dot dl through the superconducting self inductor the answer is ZERO as the E-field in the self-inductor is zero. If you place a voltmeter over the self-inductor, then you have added a loop to the circuit and you will have to apply Faraday's law in that loop. You will then find that the voltmeter reads the value + or minus L*dI/dt. I suggest you watch 2 of my lectures ruclips.net/video/LzT_YZ0xCFY/видео.html and
      ruclips.net/video/nGQbA2jwkWI/видео.html

    • @muneer332
      @muneer332 8 лет назад +1

      thank u very much sir

  • @arkadiptaghosh99
    @arkadiptaghosh99 7 лет назад

    sir at around 11 minute you wrote the value of tan fie....but I think there will be 1overR in the denominator not R...help me out if I am correct or not????

    • @arkadiptaghosh99
      @arkadiptaghosh99 7 лет назад

      Lectures by Walter Lewin. They will make you ♥ Physics. i am saying that tan of fie should have a 1 OVER R downstairs.... according to the phasor......in Z there is 1 OVER R^2

    • @lecturesbywalterlewin.they9259
      @lecturesbywalterlewin.they9259  7 лет назад +5

      my apologies, the msg I sent you yesterday was NOT meant for you but for someone else who wrote a msg about 1/R.
      Yes, you are right, tan of phi the R should be upstairs. 1/(1/R)=R.

    • @arkadiptaghosh99
      @arkadiptaghosh99 7 лет назад

      Lectures by Walter Lewin. They will make you ♥ Physics. thank you sir,,,you make me feel physics equation and see through them....you make physics a art......thank you for that professor....wish I have ever a chance to meet you.....

    • @lecturesbywalterlewin.they9259
      @lecturesbywalterlewin.they9259  7 лет назад +1

      I'll be happy to meet with you. I live in the US (East coast).

    • @arkadiptaghosh99
      @arkadiptaghosh99 7 лет назад

      Lectures by Walter Lewin. They will make you ♥ Physics. i live in India.....if I get there..... I will contact you.....sir.....I want to tell you one thing the most physics teacher should learn from you that Kirchoff's rule is a special case of Faraday's law....that is amazing...concept is stairway to mathematics,most of us do blind maths....you came to India 2 or a year ago in IIT,I lost the chance to meet you.....

  • @life_well11
    @life_well11 3 года назад

    💟💟💟

  • @shreedevianantha983
    @shreedevianantha983 3 года назад

    hope I have a chance to give a million likes

  • @lefterisspanoudakis3152
    @lefterisspanoudakis3152 Год назад

    Shouldn't be tan(φ) = R*( ωC - (1 / ωL) ) ???

    • @lecturesbywalterlewin.they9259
      @lecturesbywalterlewin.they9259  Год назад

      what I have is corect - 0fcoz -

    • @lefterisspanoudakis3152
      @lefterisspanoudakis3152 Год назад

      @@lecturesbywalterlewin.they9259 I am pretty sure that R should be in the numerator. What you have is not dimensionless as tanφ should be. From the triangle tanφ = opposite / adjacent = Ε0*(ωC - 1 / ωL) / Ε0*(1/R) = R*(ωC - 1 / ωL)

  • @AsthaTravelAdventures
    @AsthaTravelAdventures 3 года назад

    Studying all this class 12 in school: ( in India

  • @sanketpawar4253
    @sanketpawar4253 6 лет назад

    Hello sir. I think you have by mistake written that,
    I(L)t= (epsilon 0/omega)×sin(omega t) +...
    I think the L is missing her. That is it must be,
    I(L)t= (epsilon 0/omega L) × sin(omega t)+....

    • @lecturesbywalterlewin.they9259
      @lecturesbywalterlewin.they9259  6 лет назад

      how many minutes into the video?

    • @sanketpawar4253
      @sanketpawar4253 6 лет назад

      Lectures by Walter Lewin. They will make you ♥ Physics.
      Sorry for the late reply.
      It is at 5 minutes and 25 seconds.
      And love you sir.

  • @mahmouddesokey3818
    @mahmouddesokey3818 5 лет назад

    Please sir , is the voltage across these parallel branches the same at any moment in time or i will get different values for different paths ?

  • @mahmouddesokey3818
    @mahmouddesokey3818 4 года назад

    Sir , imagine we have an ideal ac voltage source in series with an ideal self inductor and we connect them by superconducting wires , will we have an induced emf in that circuit and if there is an induced emf , what will be the location of the emf ?

    • @lecturesbywalterlewin.they9259
      @lecturesbywalterlewin.they9259  4 года назад

      this is a classic question. physics.stackexchange.com/questions/108821/what-happens-in-a-circuit-when-the-wire-and-the-battery-are-superconducting-an

  • @remielsmetaverse
    @remielsmetaverse 3 года назад

    😘

  • @spiral09
    @spiral09 2 года назад

    What is he talking about 🥲